Diễn Đàn MathScopeDiễn Đàn MathScope
  Diễn Đàn MathScope
Ghi Danh Hỏi/Ðáp Community Lịch

Go Back   Diễn Đàn MathScope > Sơ Cấp > Đại Số và Lượng Giác > Các Bài Toán Đã Được Giải

News & Announcements

Ngoài một số quy định đã được nêu trong phần Quy định của Ghi Danh , mọi người tranh thủ bỏ ra 5 phút để đọc thêm một số Quy định sau để khỏi bị treo nick ở MathScope nhé !

* Nội quy MathScope.Org

* Một số quy định chung !

* Quy định về việc viết bài trong diễn đàn MathScope

* Nếu bạn muốn gia nhập đội ngũ BQT thì vui lòng tham gia tại đây

* Những câu hỏi thường gặp

* Về việc viết bài trong Box Đại học và Sau đại học


Ðề tài đã khoá Gởi Ðề Tài Mới
 
Ðiều Chỉnh Xếp Bài
Old 26-07-2011, 08:19 PM   #181
leviethai
+Thành Viên+
 
Tham gia ngày: Nov 2008
Đến từ: Thành phố Hồ Chí Minh. Nhưng quê tôi là Ninh Bình.
Bài gởi: 513
Thanks: 121
Thanked 787 Times in 349 Posts
Gửi tin nhắn qua Yahoo chát tới leviethai
Đơn giản, dễ chơi dễ trúng thưởng .

Bài toán. Cho $a,b,c $ là các số thực không âm thỏa mãn $a+b+c=1. $ Chứng minh rằng

$\[\left( {ab + \frac{c}{{a + b}}} \right)\left( {bc + \frac{a}{{b + c}}} \right)\left( {ca + \frac{b}{{c + a}}} \right) \le \frac{1}{4}.\] $

[RIGHT][I][B]Nguồn: MathScope.ORG[/B][/I][/RIGHT]
 
leviethai is offline  
Old 26-07-2011, 08:32 PM   #182
bboy114crew
+Thành Viên+
 
Tham gia ngày: Oct 2010
Đến từ: Dòng thời gian...
Bài gởi: 294
Thanks: 290
Thanked 189 Times in 91 Posts
Gửi tin nhắn qua Yahoo chát tới bboy114crew
Một bài của bác Phạm Kim Hùng!
Cho các số không âm tuỳ ý a , b ,c . Bạn hãy cmr $\frac{1}{\sqrt{4a^2 + bc}} + \frac{1}{\sqrt{4b^2 + ac}} + \frac{1}{\sqrt{4c^2 + ab}} \geq \frac{4}{a + b + c} $
[RIGHT][I][B]Nguồn: MathScope.ORG[/B][/I][/RIGHT]
 
__________________
Thay đổi tất cả và mãi mãi......
Offline...

thay đổi nội dung bởi: bboy114crew, 27-07-2011 lúc 09:07 AM
bboy114crew is offline  
Old 26-07-2011, 08:49 PM   #183
chém gà
+Thành Viên+
 
Tham gia ngày: Jul 2011
Đến từ: France
Bài gởi: 136
Thanks: 8
Thanked 60 Times in 54 Posts
Trích:
Nguyên văn bởi bboy114crew View Post
Một bài của bác Phạm Kim Hùng!
Cho các số thực dương tuỳ ý a , b ,c . Bạn hãy cmr $\frac{1}{\sqrt{4a^2 + bc}} + \frac{1}{\sqrt{4b^2 + ac}} + \frac{1}{\sqrt{4c^2 + ab}} \geq \frac{4}{a + b + c} $
Sao mình không thấy dấu bằng của BDT vậy?Sai đề hay sao ấy.
[RIGHT][I][B]Nguồn: MathScope.ORG[/B][/I][/RIGHT]
 
__________________
Gà nhất quả đất-Nát nhất Sever

chém gà is offline  
Old 26-07-2011, 09:03 PM   #184
leviethai
+Thành Viên+
 
Tham gia ngày: Nov 2008
Đến từ: Thành phố Hồ Chí Minh. Nhưng quê tôi là Ninh Bình.
Bài gởi: 513
Thanks: 121
Thanked 787 Times in 349 Posts
Gửi tin nhắn qua Yahoo chát tới leviethai
Trích:
Nguyên văn bởi chém gà View Post
Sao mình không thấy dấu bằng của BDT vậy?Sai đề hay sao ấy.
Không có dấu bằng đâu có nghĩa là sai.

Ví dụ $2>1 $ không có dấu bằng nhưng vẫn đúng .

Dấu bằng của bất đẳng thức này là $a=b,c\to 0. $
[RIGHT][I][B]Nguồn: MathScope.ORG[/B][/I][/RIGHT]
 
leviethai is offline  
Old 26-07-2011, 09:08 PM   #185
chém gà
+Thành Viên+
 
Tham gia ngày: Jul 2011
Đến từ: France
Bài gởi: 136
Thanks: 8
Thanked 60 Times in 54 Posts
Trích:
Nguyên văn bởi leviethai View Post
Không có dấu bằng đâu có nghĩa là sai.

Ví dụ $2>1 $ không có dấu bằng nhưng vẫn đúng .

Dấu bằng của bất đẳng thức này là $a=b,c\to 0. $
Ý tôi phải cho điều kiện là a,b,c không âm chứ không bảo BDT sai
[RIGHT][I][B]Nguồn: MathScope.ORG[/B][/I][/RIGHT]
 
__________________
Gà nhất quả đất-Nát nhất Sever

chém gà is offline  
Old 27-07-2011, 09:05 AM   #186
bboy114crew
+Thành Viên+
 
Tham gia ngày: Oct 2010
Đến từ: Dòng thời gian...
Bài gởi: 294
Thanks: 290
Thanked 189 Times in 91 Posts
Gửi tin nhắn qua Yahoo chát tới bboy114crew
Trích:
Nguyên văn bởi chém gà View Post
Ý tôi phải cho điều kiện là a,b,c không âm chứ không bảo BDT sai
Ui gia!Đúng là a,b,c ko âm!Mình chép sai đề,xin thứ lỗi!
Dấu = xay ra khi :
$a=b;c=0 $ và các hoán vị!
[RIGHT][I][B]Nguồn: MathScope.ORG[/B][/I][/RIGHT]
 
__________________
Thay đổi tất cả và mãi mãi......
Offline...

thay đổi nội dung bởi: bboy114crew, 27-07-2011 lúc 09:08 AM
bboy114crew is offline  
Old 27-07-2011, 09:31 AM   #187
Nguyenhuyen_AG
+Thành Viên+
 
Nguyenhuyen_AG's Avatar
 
Tham gia ngày: Apr 2010
Bài gởi: 300
Thanks: 35
Thanked 307 Times in 151 Posts
Với $a,b,c $ là các số thực dương. Chứng minh

$\sqrt[3]{\frac{a^2}{b}}+\sqrt[3]{\frac{b^2}{c}}+\sqrt[3]{\frac{c^2}{a}}\ge3\sqrt[3]{\frac{ab+bc+ca}{a+b+c}} $

[RIGHT][I][B]Nguồn: MathScope.ORG[/B][/I][/RIGHT]
 
__________________
Nguyen Van Huyen
Ho Chi Minh City University of Transport
Nguyenhuyen_AG is offline  
Old 27-07-2011, 01:16 PM   #188
daiduong1095
+Thành Viên+
 
daiduong1095's Avatar
 
Tham gia ngày: Sep 2010
Đến từ: CVP-Math
Bài gởi: 287
Thanks: 13
Thanked 210 Times in 112 Posts
Gửi tin nhắn qua Yahoo chát tới daiduong1095
Trích:
Nguyên văn bởi Nguyenhuyen_AG View Post
Với $a,b,c $ là các số thực dương. Chứng minh

$\sqrt[3]{\frac{a^2}{b}}+\sqrt[3]{\frac{b^2}{c}}+\sqrt[3]{\frac{c^2}{a}}\ge3\sqrt[3]{\frac{ab+bc+ca}{a+b+c}} $
Bổ đề:Cho a,b,c là các số thực dương.Cmr:
$27(a^2b+b^2c+c^2a)(ab+bc+ca) \le(a+b+c)^5 $


Áp dụng BDT Horder ta có ngay:
$\left(\sum \sqrt[3]{\frac{a^2}{b}} \right)\left(\sum \sqrt[3]{\frac{a^2}{b}} \right)\left(\sum \sqrt[3]{\frac{a^2}{b}} \right)(a^2b+b^2c+c^2a) \ge (a+b+c)^4 $
Do đó mà:$\left(\sum \sqrt[3]{\frac{a^2}{b}} \right)^3 \ge \frac{(a+b+c)^4}{a^2b+b^2c+c^2a} \ge \frac{27(ab+bc+ca)}{a+b+c} $
[RIGHT][I][B]Nguồn: MathScope.ORG[/B][/I][/RIGHT]
 
__________________
daiduong1095 is offline  
Old 27-07-2011, 01:16 PM   #189
khtoan
+Thành Viên+
 
Tham gia ngày: Jan 2010
Đến từ: Đà Nẵng
Bài gởi: 155
Thanks: 23
Thanked 128 Times in 68 Posts
Trích:
Nguyên văn bởi Nguyenhuyen_AG View Post
Với $a,b,c $ là các số thực dương. Chứng minh

$\sqrt[3]{\frac{a^2}{b}}+\sqrt[3]{\frac{b^2}{c}}+\sqrt[3]{\frac{c^2}{a}}\ge3\sqrt[3]{\frac{ab+bc+ca}{a+b+c}} $
Không mất tính tổng quát ta có thể thay $(a,b,c)\rightarrow (a^3,b^3,c^3) $

Khi đó,bất đẳng thức cần chứng minh tương đương với :

$\frac{a^2}{b}+\frac{b^2}{c}+\frac{c^2}{a}\geq 3\sqrt[3]{\frac{a^3b^3+b^3c^3+c^3a^3}{a^3+b^3+c^3}} $

Ta có

$\frac{a^2}{b}+\frac{b^2}{c}+\frac{c^2}{a}\geq 3\sqrt[4]{\frac{a^4+b^4+c^4}{3}}\geq 3\sqrt[3]{\frac{a^3+b^3+c^3}{3}} $


Vậy ta chỉ cần chứng minh bất đẳng thức sau là đủ :

$\sqrt[3]{\frac{a^3+b^3+c^3}{3}}\geq \sqrt[3]{\frac{a^3b^3+b^3c^3+c^3a^3}{a^3+b^3+c^3}} $ (hiển nhiên đúng theo AM-GM)

Ta có điều phải chứng minh
[RIGHT][I][B]Nguồn: MathScope.ORG[/B][/I][/RIGHT]
 
khtoan is offline  
Old 27-07-2011, 03:16 PM   #190
MathForLife
+Thành Viên+
 
Tham gia ngày: Sep 2010
Đến từ: CT force
Bài gởi: 731
Thanks: 603
Thanked 425 Times in 212 Posts
Trích:
Nguyên văn bởi leviethai View Post
Bài toán. Cho $a,b,c $ là các số thực không âm thỏa mãn $a+b+c=1. $ Chứng minh rằng

$\[\left( {ab + \frac{c}{{a + b}}} \right)\left( {bc + \frac{a}{{b + c}}} \right)\left( {ca + \frac{b}{{c + a}}} \right) \le \frac{1}{4}.\] $


Ta có:

$ab+\frac{c}{a+b}=\frac{ab(a+b)+c(a+b+c)}{a+b}= \frac{(a+c)(b+c)-abc}{a+b} $

Vậy bất đẳng thức đã cho tương đương với:

$\prod (a+b)\prod \left ( 1-\frac{abc}{(a+b)(a+c)} \right )\le \frac{1}{4} $

Với $1\ge x;y;z\ge 0 $ thì $(1-x)(1-y)(1-z)\le 1-(x+y+z)+xy+yz+zx $ . Áp dụng điều này ta có:

$\prod \left ( 1-\frac{abc}{(a+b)(a+c)} \right )\le 1-\sum \frac{abc}{(a+b)(a+c)}+\frac{a^2b^2c^2}{(a+b)(b+c) (c+a)}\sum \frac{1}{a+b} $

$\Leftrightarrow \prod (a+b)\prod \left ( 1-\frac{abc}{(a+b)(a+c)} \right )\le \sum ab(a+b)+a^2b^2c^2\sum \frac{1}{a+b}=A $

Để chứng minh $A\le \frac{1}{4} $ , ta chuyển về dạng đồng bậc:

$4\left (\sum ab(a+b)+a^2b^2c^2\sum \frac{1}{a+b}\right )\le (a+b+c)^3 $

Bất đẳng thức Schur cho ta $4\sum ab(a+b)\le (a+b+c)^3-3abc $ . Công việc còn lại là chứng minh:

$4\sum \frac{abc}{a+b}\le 3 $

Theo bất đẳng thức AM-GM:

$4\sum \frac{abc}{a+b}\le \sum a(b+c)=2(ab+bc+ca)\le \frac{2}{3}<3 $

Ta được đpcm. Đẳng thức xảy ra khi và chỉ khi $a=b=0.5;c=0 $ và các hoán vị.


[RIGHT][I][B]Nguồn: MathScope.ORG[/B][/I][/RIGHT]
 

thay đổi nội dung bởi: leviethai, 28-07-2011 lúc 01:50 PM
MathForLife is offline  
The Following User Says Thank You to MathForLife For This Useful Post:
bboy114crew (27-07-2011)
Old 27-07-2011, 04:14 PM   #191
bboy114crew
+Thành Viên+
 
Tham gia ngày: Oct 2010
Đến từ: Dòng thời gian...
Bài gởi: 294
Thanks: 290
Thanked 189 Times in 91 Posts
Gửi tin nhắn qua Yahoo chát tới bboy114crew
Cho a, b, c là các số thực dương thỏa mãn
$\frac{1}{{a + b + 1}} + \frac{1}{{b + c + 1}} + \frac{1}{{c + a + 1}} \ge 1 $
Chứng minh rằng $a+b+c \geq ab+bc+ca $
[RIGHT][I][B]Nguồn: MathScope.ORG[/B][/I][/RIGHT]
 
__________________
Thay đổi tất cả và mãi mãi......
Offline...
bboy114crew is offline  
Old 27-07-2011, 04:25 PM   #192
ptk_1411
Moderator
 
ptk_1411's Avatar
 
Tham gia ngày: Apr 2011
Bài gởi: 698
Thanks: 162
Thanked 813 Times in 365 Posts
Trích:
Nguyên văn bởi bboy114crew View Post
Cho a, b, c là các số thực dương thỏa mãn
$\frac{1}{{a + b + 1}} + \frac{1}{{b + c + 1}} + \frac{1}{{c + a + 1}} \ge 1 $
Chứng minh rằng $a+b+c \geq ab+bc+ca $
Theo bất đẳng thức Cauchy-Schwarz:

$(a+b+1)(a+b+c^2)\ge (a+b+c)^2 $

$\Rightarrow \dfrac{1}{a+b+1}\le \dfrac{a+b+c^2}{(a+b+c)^2} $

Thiết lập các bât đẳng thức tương tự rồi cộng lại, ta được:

$\dfrac{\sum a^2+2\sum a}{(a+b+c)^2}\ge \sum \dfrac{1}{a+b+1}\ge 1 $

$\Rightarrow \sum a^2+2\sum a\ge (a+b+c)^2 $

$\Rightarrow a+b+c\ge ab+bc+ca $

[RIGHT][I][B]Nguồn: MathScope.ORG[/B][/I][/RIGHT]
 
__________________
P.T.K
Có xa xôi mấy mà tình xa xôi...
ptk_1411 is offline  
Old 27-07-2011, 04:40 PM   #193
daiduong1095
+Thành Viên+
 
daiduong1095's Avatar
 
Tham gia ngày: Sep 2010
Đến từ: CVP-Math
Bài gởi: 287
Thanks: 13
Thanked 210 Times in 112 Posts
Gửi tin nhắn qua Yahoo chát tới daiduong1095
Trích:
Nguyên văn bởi bboy114crew View Post
Cho a, b, c là các số thực dương thỏa mãn
$\frac{1}{{a + b + 1}} + \frac{1}{{b + c + 1}} + \frac{1}{{c + a + 1}} \ge 1 $
Chứng minh rằng $a+b+c \geq ab+bc+ca $
Bài này bạn đã post lên một lần rồi.[Only registered and activated users can see links. ]
[RIGHT][I][B]Nguồn: MathScope.ORG[/B][/I][/RIGHT]
 
__________________
daiduong1095 is offline  
Old 27-07-2011, 05:04 PM   #194
bboy114crew
+Thành Viên+
 
Tham gia ngày: Oct 2010
Đến từ: Dòng thời gian...
Bài gởi: 294
Thanks: 290
Thanked 189 Times in 91 Posts
Gửi tin nhắn qua Yahoo chát tới bboy114crew
Trích:
Nguyên văn bởi daiduong1095 View Post
Cách 1(Andrei Ciupan):Áp dụng C-S:$(a+b+1)(a+b+c^2)\ge(a+b+c)^2 $
Suy ra:$1\le\sum \frac{1}{a+b+1} \le \sum\frac{a+b+c^2}{(a+b+c)^2} \Leftrightarrow (a+b+c)^2\le 2(a+b+c)+a^2+b^2+c^2 $ hay đpcm.

Cách 2(Cezar Lupu):Áp dụng C-S ta có:
$2\ge\sum(1-\frac{1}{a+b+1})=\sum{\frac{a+b}{a+b+1}} \ge \frac{\left(a+b+b+c+c+a \right)^2}{(a+b)(a+b+1)+(b+c)(b+c+1)+(c+a)(c+a+1)} $
$\Rightarrow a^2+b^2+c^2+ab+bc+ca+a+b+c \ge (a+b+c)^2 \Rightarrow $đpcm.

Cách 3(của anh Cẩn)Giả sử tồn tại các số dương a,b,c sao cho:
$\sum \frac{1}{a+b+1} \geq1 $ và $a+b+c<ab+bc+ca $.Khi đó ta có:
$\frac{1}{a+b+1}<\frac{\frac{ab+bc+ca}{a+b+c}}{a+b+ c+\frac{ab+bc+ca}{a+b+c}}=\frac{ab+bc+ca}{(a+b)(a+ b+c)+ab+bc+ca} $
Suy ra:$\sum{\frac{ab+bc+ca}{(a+b)(a+b+c)+ab+bc+ca}}>1 \Leftrightarrow 1>\sum(1-\frac{2(ab+bc+ca)}{(a+b)(a+b+c)+ab+bc+ca}) $
$\Leftrightarrow 1> \sum{\frac{a^2+ab+b^2}{(a+b)(a+b+c)+ab+bc+ca} \ge \frac{3}{4}\sum{\frac{(a+b)^2}{(a+b)(a+b+c)+ab+bc+ ca}} $
$\ge \frac{3(a+b+c)^2}{\sum{[(a+b)(a+b+c)+ab+bc+ca}]}=\frac{3(a+b+c)^2}{2(a+b+c)^2+3(ab+bc+ca)} \ge1 $Vô lí.
.....................đpcm.
Post by Phạm Kim Hùng!




Típ!
Cho a,b,c>0 CMR
${\left( {\frac{a}{b}} \right)^2} + {(\frac{b}{c})^2} + {(\frac{c}{a})^2} + \frac{{10(ab + bc + ca)}}{{{a^2} + {b^2} + {c^2}}} \ge 13 $
(Nguồn mathlinks.ro)
[RIGHT][I][B]Nguồn: MathScope.ORG[/B][/I][/RIGHT]
 
__________________
Thay đổi tất cả và mãi mãi......
Offline...
bboy114crew is offline  
Old 27-07-2011, 05:46 PM   #195
daiduong1095
+Thành Viên+
 
daiduong1095's Avatar
 
Tham gia ngày: Sep 2010
Đến từ: CVP-Math
Bài gởi: 287
Thanks: 13
Thanked 210 Times in 112 Posts
Gửi tin nhắn qua Yahoo chát tới daiduong1095
Cho $a,b,c>0 $ thỏa mãn $a+b+c=1 $.Cmr:
$\frac{1}{a}+\frac{1}{b}+\frac{1}{c}+48(ab+bc+ca) \ge 25 $
[RIGHT][I][B]Nguồn: MathScope.ORG[/B][/I][/RIGHT]
 
__________________
daiduong1095 is offline  
Ðề tài đã khoá Gởi Ðề Tài Mới

Bookmarks


Quuyền Hạn Của Bạn
You may not post new threads
You may not post replies
You may not post attachments
You may not edit your posts

BB code is Mở
Smilies đang Mở
[IMG] đang Mở
HTML đang Tắt

Chuyển đến


Múi giờ GMT. Hiện tại là 02:13 PM.


Powered by: vBulletin Copyright ©2000-2024, Jelsoft Enterprises Ltd.
Inactive Reminders By mathscope.org
[page compression: 108.57 k/124.61 k (12.87%)]